Complex Exam 3

Ace your homework & exams now with Quizwiz!

The client with chronic renal failure has an indwelling catheter for peritoneal dialysis in the abdomen. The client spills water on the catheter dressing while bathing. The nurse should immediately: 1. Reinforce the dressing 2. Change the dressing 3. Flush the peritoneal dialysis catheter 4. Scrub the catheter with povidone-iodine

2. Change the dressing Clients with peritoneal dialysis catheters are at high risk for infection. A dressing that is wet is a conduit for bacteria for bacteria to reach the catheter insertion site. The nurse assures that the dressing is kept dry at all times. Option A: Reinforcing the dressing is not a safe practice to prevent infection in this circumstance. Option C: Flushing the catheter is not indicated. Option D: Scrubbing the catheter with povidone-iodine is done at the time of connection or disconnecting of peritoneal dialysis.

Polystyrene sulfonate (Kayexalate) is used in renal failure to: 1. Correct acidosis 2. Reduce serum phosphate levels 3. Exchange potassium for sodium 4. Prevent constipation from sorbitol use

3. Exchange potassium for sodium In renal failure, patients become hyperkalemic because they can't excrete potassium in the urine. Polystyrene sulfonate acts to excrete potassium by pulling potassium into the bowels and exchanging it for sodium.

Which of the following definitions best describes gastritis? 1. Erosion of the gastric mucosa 2. Inflammation of a diverticulum 3. Inflammation of the gastric mucosa 4. Reflux of stomach acid into the esophagus

3. Inflammation of the gastric mucosa Gastritis is an inflammation of the gastric mucosa that may be acute (often resulting from exposure to local irritants) or chronic (associated with autoimmune infections or atrophic disorders of the stomach). Erosion of the mucosa results in ulceration. Inflammation of a diverticulum is called diverticulitis; reflux of stomach acid is known as gastroesophageal disease.

Which of the following clients is at greatest risk for developing acute renal failure? 1. A dialysis client who gets influenza 2. A teenager who has an appendectomy 3. A pregnant woman who has a fractured femur 4. A client with diabetes who has a heart catheterization

4. A client with diabetes who has a heart catheterization Clients with diabetes are prone to renal insufficiency and renal failure. The contrast used for heart catheterization must be eliminated by the kidneys, which further stresses them and may produce acute renal failure. A dialysis client already has end-stage renal disease and wouldn't develop acute renal failure. Options B and C: A teenager who has an appendectomy and a pregnant woman with a fractured femur isn't at increased risk for renal failure

Which of the following factors causes the nausea associated with renal failure? 1. Oliguria 2. Gastric ulcers 3. Electrolyte imbalances 4. Accumulation of waste products

4. Accumulation of waste products Although clients with renal failure can develop stress ulcers, the nausea is usually related to the poisons of metabolic wastes that accumulate when the kidneys are unable to eliminate them. Options A and C: The client has electrolyte imbalances and oliguria, but these don't directly cause nausea.

A client has a history of chronic renal failure and received hemodialysis treatments three times per week through an arteriovenous (AV) fistula in the left arm. Which of the following interventions is included in this client's plan of care? 1. Keep the AV fistula site dry. 2. Keep the AV fistula wrapped in gauze. 3. Take the blood pressure in the left arm 4. Assess the AV fistula for a bruit and thrill

4. Assess the AV fistula for a bruit and thrill Assessment of the AV fistula for bruit and thrill is important because, if not present, it indicates a non-functioning fistula. Option A: When not being dialyzed, the AV fistula site may get wet. Option B: Immediately after a dialysis treatment, the access site is covered with adhesive bandages. Option C: No blood pressures or venipunctures should be taken in the arm with the AV fistula.

Rob is a 46 y.o. admitted to the hospital with a suspected diagnosis of Hepatitis B. He's jaundiced and reports weakness. Which intervention will you include in his care? 1. Regular exercise. 2. A low-protein diet. 3. Allow patient to select his meals. 4. Rest period after small, frequent meals.

4. Rest period after small, frequent meals. Rest periods and small frequent meals are indicated during the acute phase of hepatitis B.

A patient diagnosed with viral hepatitis is in the pre-icteric phase. When assessing the patient, which of these findings should the healthcare provider anticipate? (Select all that apply) A.Nausea B. Pruritis C. Tarry stools D. Anorexia E. Dark urine

A and D

A patient diagnosed with viral hepatitis develops liver failure and hepatic encephalopathy. Which of these measures should the healthcare provider include in this patient's plan of care? (select all that apply) A. Monitor the patient's blood glucose B. Monitor the patient's protime (PT) C. Institute droplet precautions D. Assess deep tendon reflexes E. Provide high-protein feedings

A, B, D

A patient diagnosed with chronic hepatitis has developed hepatic encephalopathy. When assessing the patient, the healthcare provider looks for which of the following clinical manifestations characteristic of this condition (Select all that apply) A. Slurred speech B. Bloody emesis C. Retroperitoneal bleeding D. Shortened attention span E. Hypersomnia F. Involuntary hand tremor

A, D, E, F

You're providing an in-service to new nurse graduates about esophageal varices in patients with cirrhosis. You ask the graduates to list activities that should be avoided by a patient with this condition. Which activities listed are correct: Select all that apply A. Excessive coughing B. Sleeping on the back C. Drinking juice D. Alcohol consumption E. Straining during a bowel movement F. Vomiting

A, D, E, F Esophageal varices are dilated vessels that are connected from the throat to the stomach. They can become enlarged due to portal hypertension in cirrhosis and can rupture (this is a medical emergency). The patient should avoid activities that could rupture these vessels, such as excessive cough, vomiting, drinking alcohol, and constipation (straining increases thoracic pressure.)

A female client who has just been diagnosed with hepatitis A asks, "How could I have gotten this disease?" What is the nurse's best response? A. "You may have eaten contaminated restaurant food." B. "You could have gotten it by using I.V. drugs." C. "You must have received an infected blood transfusion." D. "You probably got it by engaging in unprotected sex."

A. "You may have eaten contaminated restaurant food." Hepatitis A virus typically is transmitted by the oral-fecal route — commonly by consuming food contaminated by infected food handlers. The virus isn't transmitted by the I.V. route, blood transfusions, or unprotected sex. Hepatitis B can be transmitted by I.V. drug use or blood transfusion. Hepatitis C can be transmitted by unprotected sex.

A nurse observes the client receiving fat emulsions is having hives. A nurse reviews the client's history and note in which of the following may cause about by the complaint of the client? A. Allergy to an egg. B. Allergy to peanut. C. Allergy to shellfish. D. Allergy to corn

A. Allergy to an egg. Fat emulsions (lipids) contain egg yolk phospholipids and should not be given to clients with egg allergies.

A nurse is teaching a client with pancreatitis about following a low-fat diet. The nurse develops a list of high-fat foods to avoid and includes which food on the item list? A. Chocolate milk. B. Broccoli. C. Apple. D. Salmon.

A. Chocolate milk. Chocolate milk is a high-fat food. Options B and C: Fruits and vegetables are low in fat because they do not come from animal sources. Option D: Salmon is naturally lower in fat.

An allergy to eggs or soybeans may indicate a risk for reaction to which of the following? A. Propofol B. Povidone-Iodine C. Latex D. Shellfish

A. Propofol

While assisting a patient with chronic pancreatitis to the bathroom, you note the patient's stool to be oily/greasy in appearance. In your documentation you note this as: A. Steatorrhea B. Melena C. Currant D. Hematochezia

A. Steatorrhea Steatorrhea is an oily/greasy appearance of the stool which can occur in chronic pancreatitis. This occurs due to the inability of the pancreas to produce digestive enzymes which help break down fats. Fats are not being broken down; therefore, it is being excreted into the stool. Melena is used to describe tarry/black stool, hematochezia is used to describe red stools, and currant are jelly type stools.

A patient is diagnosed with an infection caused by the hepatitis A virus. Which statement, if made by the patient, would indicate the patient needs further teaching about the infection? A. "It's important for me to remember to wash my hands after I use the bathroom." B. "I might get liver cancer someday because I have this infection." C. "Before I take any over-the-counter medicines I should call the clinic." D. "I will wash raw fruits and vegetables thoroughly before I eat them."

B. "I might get liver cancer someday because I have this infection."

A patient who received treatment for pancreatitis is being discharged home. You're providing diet teaching to the patient. Which statement by the patient requires immediate re-education about the diet restrictions? A. "It will be hard but I will eat a diet low in fat and avoid greasy foods." B. "It is very important I limit my alcohol intake to no more than 2-3 glasses of wine a week." C. "I will concentrate on eating complex carbohydrates rather than refined carbohydrates." D. "I will purchase foods that are high in protein."

B. "It is very important I limit my alcohol intake to no more than 2-3 glasses of wine a week." A patient with pancreatitis should AVOID any amount of alcohol because of its effects on the pancreas. Remember alcohol is a cause of both acute and chronic pancreatitis. All the other options are correct.

Which of the following will the nurse include in the care plan for a client hospitalized with viral hepatitis? A. Increase fluid intake to 3000 ml per day B. Adequate bed rest C. Bland diet D. Administer antibiotics as ordered

B. Adequate bed rest Treatment of hepatitis consists of bed rest during the acute phase to reduce metabolic demands on the liver, thus increasing blood supply and cell regeneration. Forcing fluids, antibiotics, and bland diets are not part of the treatment plan for viral hepatitis.

A patient with CKD has a low erythropoietin (EPO) level. The patient is at risk for? A. Hypercalcemia B. Anemia C. Blood clots D. Hyperkalemia

B. Anemia EPO (erythropoietin) helps create red blood cells in the bone marrow. The kidneys produce EPO and when the kidneys are damaged in CKD they can decrease in the production of EPO. Therefore, the patient is at risk for anemia.

For a client with hepatic cirrhosis who has altered clotting mechanisms, which intervention would be most important? A. Allowing complete independence of mobility B. Applying pressure to injection sites C. Administering antibiotics as prescribed D. Increasing nutritional intake

B. Applying pressure to injection sites The client with cirrhosis who has altered clotting is at high risk for hemorrhage. Prolonged application of pressure to injection or bleeding sites is important. Complete independence may increase the client's potential for injury, because an unsupervised client may injure himself and bleed excessively. Antibiotics and good nutrition are important to promote liver regeneration. However, they are not most important for a client at high risk for hemorrhage.

The nurse is instructing a client with hyperkalemia on the importance of choosing foods low in potassium. The nurse should teach the client to limit which of the following foods? A. Grapes. B. Carrot. C. Green beans. D. Lettuce.

B. Carrot. Carrots has 320 mg of potassium per 100 mg serving; green beans give 209 mg of potassium, 194 mg for lettuce, and 191 mg for grapes all in 100 mg serving. Other foods that are low in potassium include: applesauce, blueberries, pineapple, and cabbage.

For Rico who has chronic pancreatitis, which nursing intervention would be most helpful? A. Allowing liberalized fluid intake B. Counseling to stop alcohol consumption C. Encouraging daily exercise D. Modifying dietary protein

B. Counseling to stop alcohol consumption Chronic pancreatitis typically results from repeated episodes of acute pancreatitis. More than half of chronic pancreatitis cases are associated with alcoholism. Counseling to stop alcohol consumption would be the most helpful for the client. Dietary protein modification is not necessary for chronic pancreatitis. Daily exercise and liberalizing fluid intake would be helpful but not the most beneficial intervention

A client is suspected of having hepatitis. Which diagnostic test result will assist in confirming this diagnosis? A. Elevated hemoglobin level B. Elevated serum bilirubin level C. Elevated blood urea nitrogen level D. Decreased erythrocyte sedimentation rate

B. Elevated serum bilirubin level Laboratory indicators of hepatitis include elevated liver enzyme levels, elevated serum bilirubin levels, elevated erythrocyte sedimentation rates, and leukopenia. An elevated blood urea nitrogen level may indicate renal dysfunction. A hemoglobin level is unrelated to this diagnosis.

What laboratory finding is the primary diagnostic indicator for pancreatitis? A. Elevated blood urea nitrogen (BUN) B. Elevated serum lipase C. Elevated aspartate aminotransferase (AST) D. Increased lactate dehydrogenase (LD)

B. Elevated serum lipase Elevation of serum lipase is the most reliable indicator of pancreatitis because this enzyme is produced solely by the pancreas. A client's BUN is typically elevated in relation to renal dysfunction; the AST, in relation to liver dysfunction; and LD, in relation to damaged cardiac muscle.

Lorazepam (Ativan) 1 mg IV is ordered for a 45-year-old male patient before a scheduled surgery. Which of the following is the most appropriate action for the nurse to take before the administration of this medication? A. Ask the patient about an allergy to iodine or shellfish. B. Encourage or assist the patient to the bathroom to void. C. Explain that the medication is used to prevent postoperative nausea. D. Check the laboratory results for the most recent serum potassium level.

B. Encourage or assist the patient to the bathroom to void

While providing mouth care to a patient with late-stage cirrhosis, you note a pungent, sweet, musty smell to the breath. This is known as: A. Metallic Hepatico B. Fetor Hepaticus C. Hepaticoacidosis D. Asterixis

B. Fetor Hepaticus

While assessing morning labs on your patient with CKD. You note the patient's phosphate level is 6.2 mg/dL. As the nurse, you expect to find the calcium level to be? A. Elevated B. Low C. Normal D. Same as the phosphate level

B. Low A normal phosphate level is 2.7-4.5 mg/dL. This patient is experiencing HYPERphosphatemia. When hyperphosphatemia presents the calcium level DECREASES because phosphate and calcium bind to each. When there is too much phosphate in the blood it takes too much calcium with it and it decreases the calcium in the blood. Therefore, the nurse would expect to find the calcium level decreased.

A nurse is monitoring the status of a client's fat emulsion (lipid) infusion and notes that the infusion is 2 hours delay. The nurse should do which of the following actions? A. Adjust the infusion rate to catch up over the next hour. B. Make sure the infusion rate is infusing at the ordered rate. C. Increase the infusion rate to catch up over the next few hours. D. Adjust the infusion rate to full blast until the solution is back on time.

B. Make sure the infusion rate is infusing at the ordered rate. The nurse should maintain the prescribed rate of a fat emulsion even if the infusion's time consume is behind. Options A, C, and D are incorrect since increasing the rate will potentially cause a fluid overload.

An allergy to shellfish may indicate a risk for reaction to which of the following? A. Propofol B. Povidone-Iodine C. Latex D. Soybean

B. Povidone-Iodine

A patient with Stage 5 CKD is experiencing extreme pruritus and has several areas of crystallized white deposits on the skin. As the nurse, you know this is due to excessive amounts of what substance found in the blood? A. Calcium B. Urea C. Phosphate D. Erythropoietin

B. Urea This patient is experiencing uremic frost that occurs in severe chronic kidney disease. This is due to high amounts of urea in the blood being secreted via the sweat glands onto the skin, which will appear as white deposits on the skin. The patient will experience itching with this as well.

The nurse is caring for a male client with a diagnosis of chronic gastritis. The nurse monitors the client knowing that this client is at risk for which vitamin deficiency? A. Vitamin A B. Vitamin B12 C. Vitamin C D. Vitamin E

B. Vitamin B12 Chronic gastritis causes deterioration and atrophy of the lining of the stomach, leading to the loss of the function of the parietal cells. The source of the intrinsic factor is lost, which results in the inability to absorb vitamin B12. This leads to the development of pernicious anemia. The client is not at risk for vitamin A, C, or E deficiency.

A male client has just been diagnosed with hepatitis A. On assessment, the nurse expects to note: A. severe abdominal pain radiating to the shoulder. B. anorexia, nausea, and vomiting. C. eructation and constipation. D. abdominal ascites.

B. anorexia, nausea, and vomiting. Hallmark signs and symptoms of hepatitis A include anorexia, nausea, vomiting, fatigue, and weakness. Abdominal pain may occur but doesn't radiate to the shoulder. Eructation and constipation are common in gallbladder disease, not hepatitis A. Abdominal ascites is a sign of advanced hepatic disease, not an early sign of hepatitis A.

Nurse Farrah is providing care for Kristoff who has jaundice. Which statement indicates that the nurse understands the rationale for instituting skin care measures for the client? A. "Jaundice is associated with pressure ulcer formation." B. "Jaundice impairs urea production, which produces pruritus." C. "Jaundice produces pruritus due to impaired bile acid excretion." D. "Jaundice leads to decreased tissue perfusion and subsequent breakdown."

C. "Jaundice produces pruritus due to impaired bile acid excretion." Jaundice is a symptom characterized by increased bilirubin concentration in the blood. Bile acid excretion is impaired, increasing the bile acids in the skin and causing pruritus. Jaundice is not associated with pressure ulcer formation. However, edema and hypoalbuminemia are. Jaundice itself does not impair urea production or lead to decreased tissue perfusion.

A patient diagnosed with celiac disease tells the healthcare provider, "I've been on a gluten-free diet for 3 months but this week I have had diarrhea, bloating, and gas." Which response would be most appropriate for the healthcare provider to make? A. "We'll need to take a biopsy of your small intestine to see if your disease is progressing." B. "Try adding more fiber to your daily diet and see if that resolves the problem" C. "Let's sit down and make a list of all the foods you ate this week." D. "You'll need to be compliant with your diet if you want to avoid these problems."

C. "Let's sit down and make a list of all the foods you ate this week."

Continuous type of feedings is administered over a __ hour period.? A. 4. B. 12. C. 24. D. 36.

C. 24. Continuous feeding is administered for 24 hours. An infusion pump regulates the flow

Nurse Joy is preparing to administer medication through a nasogastric tube that is connected to suction. To administer the medication, the nurse would: A. Position the client supine to assist in medication absorption B. Aspirate the nasogastric tube after medication administration to maintain patency C. Clamp the nasogastric tube for 30 minutes following administration of the medication D. Change the suction setting to low intermittent suction for 30 minutes after medication administration

C. Clamp the nasogastric tube for 30 minutes following administration of the medication If a client has a nasogastric tube connected to suction, the nurse should wait up to 30 minutes before reconnecting the tube to the suction apparatus to allow adequate time for medication absorption. Aspirating the nasogastric tube will remove the medication just administered. Low intermittent suction also will remove the medication just administered. The client should not be placed in the supine position because of the risk for aspiration.

A nurse is changing the central line dressing of a client receiving parenteral nutrition (PN) and notes that there are redness and drainage at the insertion site. The nurse next assesses which of the following? A. Time of last dressing change. B. Allergy. C. Client's temperature. D. Expiration date.

C. Client's temperature. Redness at the catheter insertion site is a possible sign of infection. The nurse would next assess for other signs of infection. Of the options given, the temperature is the next item to assess.

An allergy to bananas or kiwi may indicate a risk for reaction to which of the following? A. Propofol B. Povidone-Iodine C. Latex D. Soybean

C. Latex

A patient diagnosed with hepatitis and cirrhosis has developed ascites. When assessing the patient, the healthcare provider notes an increased temperature and a decreased level of consciousness. What assessment should the healthcare provider perform next? A. Measure the patient's abdominal girth B. Auscultate the patient's lung sounds C. Obtain a urine sample for laboratory analysis D. Palpate the abdomen for tenderness

C. Obtain a urine sample for laboratory analysis

A nurse is preparing to remove a nasogastric tube from a female client. The nurse should instruct the client to do which of the following just before the nurse removes the tube? A. Exhale B. Inhale and exhale quickly C. Take and hold a deep breath D. Perform a Valsalva maneuver

C. Take and hold a deep breath When the nurse removes a nasogastric tube, the client is instructed to take and hold a deep breath. This will close the epiglottis. This allows for easy withdrawal through the esophagus into the nose. The nurse removes the tube with one smooth, continuous pull.

Which of the following accurately describes the greatest risk related to having a feeding tube? A) Electrolyte imbalance B) Fluid volume overload C) Infection D) Aspiration

D) Aspiration

You are providing education to a patient with CKD about calcium carbonate. Which statement by the patient demonstrates they understood your teaching about this medication? A. "This medication will help keep my calcium level normal." B. "I will not take this medication with meals" C. "It is important I consume high amounts of oatmeal, poultry, fish, and dairy products while taking this medication." D. "This medication will help prevent my phosphate level from increasing."

D. "This medication will help prevent my phosphate level from increasing."

A nurse is checking the nasogastric tube position of a client receiving a long term therapy of Omeprazole (Prisolec) by aspirating the stomach contents to check for the PH level. The nurse proves that correct tube placement if the PH level is? A. 7.75. B. 7.5. C. 6.5. D. 5.5.

D. 5.5. Gastric placement is indicated by a pH of less than 4, but may increase to between pH 4-6 if the patient is receiving acid-inhibiting drugs.

A patient is admitted to the ER with the following signs and symptoms: very painful mid-epigastric pain felt in the back, elevated glucose, fever, and vomiting. During the head-to-toe assessment, you notice bluish discoloration around the belly button. As the nurse, you know this is called? A. Turner's Sign B. McBurney's Sign C. Homan's Sign D. Cullen's Sign

D. Cullen's Sign This is known as Cullen's Sign. It represents retroperitoneal bleeding from the leakage of digestive enzymes from the inflamed pancreas into the surrounding tissues which is causing bleeding and it is leaking down to umbilicus tissue. Remember the C in Cullen for "circle" and the belly button forms a circle. The patient can also have Turner's Sign which is a bluish discoloration at the flanks (side of the abdomen). Remember this by TURNER ("turn her" over on her side) which is where the bluish discoloration will be.

A patient is admitted to the medical unit with a diagnosis of hepatitis. When preparing to administer intravenous medications, the healthcare provider understands that the patient's diagnosis primarily impacts which phase of pharmacokinetics? A. Distribution B. Excretion C. Absorption D. Metabolism

D. Metabolism

A nurse is caring a client who disconnected the tubing of the parenteral nutrition from the central line catheter. A nurse suspects an occurrence of an air embolism. Which of the following is an appropriate position for the client in this kind of situation? A. On the right side, with head higher than the feet. B. On the right side, with head lower than the feet. C. On the left side, with the head higher than the feet. D. On the left side, with head lower than the feet

D. On the left side, with head lower than the feet. Air embolism happens because of the entry of air into the catheter system. If it occurs, the client should be placed in a left-side-lying position with the head be lower than the feet. This position will lessen the effect of the air traveling as a bolus to the lungs by trapping it on the right side of the heart.

Before feeding a client via NGT, the nurse checks for residual and obtains a residual amount of 90ml. What is the appropriate action for the nurse to take? A. Discard the residual amount. B. Hold the due feeding. C. Skip the feeding and administer the next feeding due in 4 hours. D. Reinstill the amount and continue with administering the feeding.

D. Reinstill the amount and continue with administering the feeding. If the residual feeding is less than 100ml, feeding is administered.

A nurse is caring for a combative client who is ordered to have a nutritional therapy using parenteral nutrition (PN). The nurse should plan which of the following measures to prevent the client from injury? A. Monitor blood glucose twice a day. B. Instruct the relative to stay with the nurse. C. Measure 24-hour intake and output. D. Secure all connections in the parenteral system.

D. Secure all connections in the parenteral system. The nurse should plan to secure all connections in the tubing. This will prevent the client from pulling the connections apart.

The physician orders a patient with pancreatitis to take a pancreatic enzyme. What assessment finding demonstrates the pancreatic enzymes are working properly?* A. Abdominal girth is decreased B. Skin turgor is less than 2 seconds C. Blood glucose is 250 D. Stools appear formed and solid

D. Stools appear formed and solid Pancreatic enzymes help the body break down carbs, proteins, and fats because the body is not sufficiently producing digestive enzymes anymore. Hence, the stool will not appear as oily or greasy (decrease in steatorrhea) but appear solid and formed.

A nurse is conducting a follow-up home visit to a client who has been discharged with a parenteral nutrition(PN). Which of the following should the nurse most closely monitor in this kind of therapy? A. Blood pressure and temperature. B. Blood pressure and pulse rate. C. Height and weight. D. Temperature and weight.

D. Temperature and weight. The client's temperature is monitored to identify signs of infection which is one of the complications of this therapy. While the weight is monitored to detect hypervolemia and to determine the effectiveness of this nutritional therapy.

A patient scheduled for hip replacement surgery in the early afternoon is NPO but receives and ingests a breakfast tray with clear liquids on the morning of surgery. What response does the nurse expect when the anesthesia care provider is notified? a. Surgery will be done as scheduled. b. Surgery will be rescheduled for the following day. c. Surgery will be postponed for 8 hours after the fluid intake. d. A nasogastric tube will be inserted to remove the fluids from the stomach.

a. Surgery will be done as scheduled. The preoperative fasting recommendations of the American Society of Anesthesiology indicate that clear liquids may be taken up to 2 hours before surgery for healthy patients undergoing elective procedures. There is evidence that longer fasting is not necessary.

A patient who has never had any prior surgeries tells the nurse doing the preoperative assessment about an allergy to bananas and avocados. Which action is most important for the nurse to take? a. Notify the dietitian about the food allergies. b. Alert the surgery center about a possible latex allergy. c. Reassure the patient that all allergies are noted on the medical record. d. Ask whether the patient uses antihistamines to reduce allergic reactions.

b. Alert the surgery center about a possible latex allergy.

The nurse is monitoring a client receiving peritoneal dialysis and nurse notes that a client's outflow is less than the inflow. Select actions that the nurse should take. (select all that apply) 1. Place the client in good body alignment 2. Check the level of the drainage bag 3. Contact the physician 4. Check the peritoneal dialysis system for kinks 5. Reposition the client to his or her side

1, 2, 4, 5. If outflow drainage is inadequate, the nurse attempts to stimulate outflow by changing the client's position. Turning the client to the other side or making sure that the client is in good body alignment may assist with outflow drainage. The drainage bag needs to be lower than the client's abdomen to enhance gravity drainage. The connecting tubing and the peritoneal dialysis system is also checked for kinks or twisting and the clamps on the system are checked to ensure that they are open. There is no reason to contact the physician.

The most common early sign of kidney disease is: 1. Sodium retention 2. Elevated BUN level 3. Development of metabolic acidosis 4. Inability to dilute or concentrate urine

2. Elevated BUN level Increased BUN is usually an early indicator of decreased renal function.

Develop a teaching care plan for Angie who is about to undergo a liver biopsy. Which of the following points do you include? 1. "You'll need to lie on your stomach during the test." 2. "You'll need to lie on your right side after the test." 3. "During the biopsy you'll be asked to exhale deeply and hold it." 4. "The biopsy is performed under general anesthesia."

2. "You'll need to lie on your right side after the test." After a liver biopsy, the patient is placed on the right side to compress the liver and to reduce the risk of bleeding or bile leakage.

A group of nursing students are studying together. They are discussing the differences between parenteral and enteral nutrition. Which statement, if made by one of the students, indicates further instruction is needed? A) "Parenteral nutrition is the administration of nutrients directly into the GI tract by way of a feeding tube." B) "Enteral nutrition is preferred because it is less expensive than parenteral nutrition and maintains functioning of the gut." C) "An example of the parenteral route is subcutaneous or IM injections, or the IV route." D) "Gastric feedings may be given to patients with a low risk of aspiration. If there is a risk of aspiration, jejunal feeding is the preferred method. Parenteral nutrition is provided if the patient's GI tract is nonfunctional."

A) "Parenteral nutrition is the administration of nutrients directly into the GI tract by way of a feeding tube."

What type of procedural information should be given to a patient in preparation for ambulatory surgery (select all that apply)? a. How pain will be controlled b. Any fluid and food restrictions c. Characteristics of monitoring equipment d. What odors and sensations may be experienced e. Technique and practice of coughing and deep breathing, if appropriate

A, B, E Procedural information includes what will or should be done for surgical preparation, including what to bring and what to wear to the surgery center, length and type of food and fluid restrictions, physical preparation required, pain control, need for coughing and deep breathing (if appropriate), and procedures done before and during surgery (such as vital signs, IV lines, and how anesthesia is administered). The other options are sensory and process information (see Table 18-6).

Your patient with cirrhosis has severe splenomegaly. As the nurse you will make it priority to monitor the patient for signs and symptoms of? Select all that apply: A. Thrombocytopenia B. Vision changes C. Increased PT/INR D. Leukopenia

A, C, D A patient with an enlarged spleen (splenomegaly) due to cirrhosis can experience thrombocytopenia (low platelet count), increased PT/INR (means it takes the patient a long time to stop bleeding), and leukopenia (low white blood cells). The spleen stores platelets and WBCs. An enlarged spleen can develop due to portal hypertension, which causes the platelets and WBCs to become stuck inside the spleen due to the increased pressure in the hepatic vein (hence lowering the count and the body's access to these important cells for survival).

Mr. Hasakusa is in end-stage liver failure. Which interventions should the nurse implement when addressing hepatic encephalopathy? (Select all that apply.) A. Assessing the client's neurologic status every 2 hours B. Monitoring the client's hemoglobin and hematocrit levels C. Evaluating the client's serum ammonia level D. Monitoring the client's handwriting daily E. Preparing to insert an esophageal tamponade tube F. Making sure the client's fingernails are short

A, C, D Hepatic encephalopathy results from an increased ammonia level due to the liver's inability to covert ammonia to urea, which leads to neurologic dysfunction and possible brain damage. The nurse should monitor the client's neurologic status, serum ammonia level, and handwriting. Monitoring the client's hemoglobin and hematocrit levels and insertion of an esophageal tamponade tube address esophageal bleeding. Keeping fingernails short address jaundice.

The healthcare provider is teaching a patient diagnosed with hepatitis C about the disease. Which of these statements made by the patient indicate that the patient has an understanding of the teaching? (Select all that apply) A. "It's important for me to use barrier protection when I have sex." B. "I'll plan to do all my activities in the morning when I'm most rested." C. "I should avoid sharing drinking cups and eating utensils with my family." D. "I should not drink any wine, beer or other alcoholic beverages." E. "Acetaminophen is the best medication for me if I have a headache." F. "I should get vaccinated for hepatitis A and hepatitis B."

A, D, F

A patient diagnosed with hepatitis is undergoing a liver biopsy. When caring for the patient, which of these actions would be essential for the healthcare provider to take? (select all that apply) A. Ensure the patient has an empty bladder before the procedure B. Provide a mechanical soft diet for before the procedure C. Review the patient's baseline liver function tests D. Help the patient assume a left lateral position after the procedure E. Monitor the patient's vital signs after the procedure F. Ensure the patient's clotting profile is within normal limits

A, E, F

A patient is admitted with hepatic encephalopathy secondary to cirrhosis. Which meal option selection below should be avoided with this patient? A. Beef tips and broccoli rabe B. Pasta noodles and bread C. Cucumber sandwich with a side of grapes D. Fresh salad with chopped water chestnuts

A. Beef tips and broccoli rabe Patients who are experiencing hepatic encephalopathy are having issues with toxin build up in the body, specifically ammonia. Remember that ammonia is the byproduct of protein breakdown, and normally the liver can take the ammonia from the protein breakdown and turn it into urea (but if the cirrhosis is severe enough this can't happen). Therefore, the patient should consume foods LOW in protein until the encephalopathy subsides. Option A is very high in protein while the others are low in protein. Remember meats, legumes, eggs, broccoli rabe, certain grains etc. are high in protein.

A patient receiving dialysis should avoid what type of foods? A. Canned soups, cold cut sandwiches, and Chinese take-out B. Fresh fruits and vegetable, poultry, and beans C. Steamed broccoli, broiled mackerel, and artificial sweeteners D. Microwaved sweet potatoes, boiled cabbage, and artichokes

A. Canned soups, cold cut sandwiches, and Chinese take-out Patients who are receiving dialysis have renal disease and therefore should follow a sodium restricted diet. Canned soups, cold cut sandwiches, and Chinese take-out are all high in sodium.

A patient with stage 4 chronic kidney disease asks what type of diet they should follow. You explain the patient should follow a: A. Low protein, low sodium, low potassium, low phosphate diet B. High protein, low sodium, low potassium, high phosphate diet C. Low protein, high sodium, high potassium, high phosphate diet D. Low protein, low sodium, low potassium, high phosphate diet

A. Low protein, low sodium, low potassium, low phosphate diet The patient should follow this type of diet because protein breaks down into urea (remember patients will have increased urea levels), low sodium to prevent fluid retention, low potassium to prevent hyperkalemia (remember glomerulus isn't filtering out potassium/phosphate as it should), and low phosphate to prevent hyperphosphatemia.

Which assessment finding indicates that lactulose is effective in decreasing the ammonia level in the client with hepatic encephalopathy? A. Passage of two or three soft stools daily B. Evidence of watery diarrhea C. Daily deterioration in the client's handwriting D. Appearance of frothy, foul-smelling stools

A. Passage of two or three soft stools daily Lactulose reduces serum ammonia levels by inducing catharsis, subsequently decreasing colonic pH and inhibiting fecal flora from producing ammonia from urea. Ammonia is removed with the stool. Two or three soft stools daily indicate effectiveness of the drug. Watery diarrhea indicates overdose. Daily deterioration in the client's handwriting indicates an increase in the ammonia level and worsening of hepatic encephalopathy. Frothy, foul-smelling stools indicate steatorrhea, caused by impaired fat digestion.

The healthcare provider is teaching a patient diagnosed with celiac disease about the disease process. Which of the following statements made by the patient would indicate a correct understanding of the teaching? A. "I have an allergy to the proteins that are found in wheat." B. "My immune system reacts to gluten and damages my gut." C. "The bacteria in my gut are not able to ferment the gluten." D. "I'm glad that I can still eat bread made with rye flour."

B. "My immune system reacts to gluten and damages my gut."

You are receiving shift report on a patient with cirrhosis. The nurse tells you the patient's bilirubin levels are very high. Based on this, what assessment findings may you expect to find during your head-to-toe assessment? Select all that apply: A. Frothy light-colored urine B. Dark brown urine C. Yellowing of the sclera D. Dark brown stool E. Jaundice of the skin F. Bluish mucous membranes

B, C, E High bilirubin levels are because the hepatocytes are no longer able to properly conjugate the bilirubin because they are damaged. This causes bilirubin to leak into the blood and urine (rather than entering the bile and being excreted in the stool). Therefore, the bilirubin stays in the blood and will enter the urine. This will cause the patient to experience yellowing of the skin, sclera of the eyes, and mucous membranes ("jaundice") and have dark brown urine. The stools would be CLAY-COLORED not dark brown (remember bilirubin normally gives stool it brown color but it will be absent).

A patient with late-stage cirrhosis develops portal hypertension. Which of the following options below are complications that can develop from this condition? Select all that apply: A. Increase albumin levels B. Ascites C. Splenomegaly D. Fluid volume deficient E. Esophageal varices

B, C, E Portal Hypertension is where the portal vein becomes narrow due to scar tissue in the liver, which is restricting the flow of blood to the liver. Therefore, pressure becomes increased in the portal vein and affects the organs connected via the vein to the liver. The patient may experience ascites, enlarged spleen "splenomegaly", and esophageal varices etc.

The physician orders Lactulose 30 mL by mouth per day for a patient with cirrhosis. What findings below demonstrates the medication is working effectively? Select all that apply: A. Decrease albumin levels B. Decrease in Fetor Hepaticus C. Patient is stuporous. D. Decreased ammonia blood level E. Presence of asterixis

B, D A patient with cirrhosis may experience a complication called hepatic encephalopathy. This will cause the patient to become confused (they may enter into a coma), have pungent, musty smelling breath (fetor hepaticus), asterixis (involuntary flapping of the hands) etc. This is due to the buildup of ammonia in the blood, which affects the brain. Lactulose can be prescribed to help decrease the ammonia levels. Therefore, if the medication is working properly to decrease the level of ammonia the patient would have improving mental status (NOT stuporous), decreased ammonia blood level, decreasing or absence of asterixis, and decreased ammonia blood level.

A client diagnosed with chronic cirrhosis who has ascites and pitting peripheral edema also has hepatic encephalopathy. Which of the following nursing interventions are appropriate to prevent skin breakdown? (Select all that apply.) A. Range of motion every 4 hours B. Turn and reposition every 2 hours C. Abdominal and foot massages every 2 hours D. Alternating air pressure mattress E. Sit in chair for 30 minutes each shift

B, D Edematous tissue must receive meticulous care to prevent tissue breakdown. Range of motion exercises preserve joint function but do not prevent skin breakdown. Abdominal or foot massage will not prevent skin breakdown but must be cleansed carefully to prevent breaks in skin integrity. The feet should be kept at the level of heart or higher so Fowler's position should be employed. An air pressure mattress, careful repositioning can prevent skin breakdown.

Assessment findings for a patient diagnosed with alcoholic hepatitis and portal hypertension include oliguria and increasing blood urea nitrogen (BUN). Which additional assessment finding would be consistent with this diagnosis? A. Hypotension and pallor B. Increased serum creatinine C. Increased urine sodium D. Flank pain and proteinuria

B. Increased serum creatinine

Which patient below is at MOST risk for CHRONIC pancreatitis? A. A 25 year old female with a family history of gallstones. B. A 35 year old male who reports social drinking of alcohol. C. A 15 year old female with cystic fibrosis. D. A 66 year old female with stomach cancer.

C. A 15 year old female with cystic fibrosis. Patients in options A and B are at slight risk for ACUTE pancreatitis not chronic. Remember the main causes of ACUTE pancreatitis are gallstones and alcohol consumption. In option C, the patient with cystic fibrosis is at MAJOR risk for CHRONIC pancreatitis because they are lacking the protein CFTR which plays a role in the movement of chloride ions to help balance salt and water in the epithelial cells that line the ducts of the pancreas. There is a decreased production of bicarbonate secretion by the epithelial cells. Therefore, this leads to thick mucus in the pancreatic ducts that can lead to blockage of the pancreatic ducts which can cause the digestive enzymes to activate and damage the pancreas. Overtime, the pancreas will experience fibrosis of the pancreas' tissue and will no longer produce digestive enzyme to help with food digestion.

Which patient below is NOT at risk for developing chronic kidney disease? A. A 58 year old female with uncontrolled hypertension. B. A 69 year old female with diabetes mellitus. C. A 45 year old female with polycystic ovarian disease. D. A 78 year old female with an intrarenal injury.

C. A 45 year old female with polycystic ovarian disease.

A client with advanced cirrhosis has been diagnosed with hepatic encephalopathy. The nurse expects to assess for: A. Malaise B. Stomatitis C. Hand tremors D. Weight loss

C. Hand tremors Hepatic encephalopathy results from the accumulation of neurotoxins in the blood, therefore the nurse wants to assess for signs of neurological involvement. Flapping of the hands (asterixis), changes in mentation, agitation, and confusion are common. These clients typically have ascites and edema so experience weight gain. Malaise and stomatitis are not related to neurological involvement.

The nurse is doing a preoperative assessment on a male patient who has type 2 diabetes mellitus, weighs 146 kg, and is 5 feet 8 inches tall. Which patient assessment is a priority related to anesthesia? a. Has hemoglobin A1C of 8.5% b. Has several seasonal allergies c. Has body mass index of 48.8 kg/m2 d Has history of postoperative vomiting

C. Has body mass index of 48.8 kg/m2 The patient's body mass index is the priority because it indicates the patient is severely obese. -The patient's size may impair the anesthesiologist's ability to ventilate and medicate the patient properly, as well as the surgery room staff's ability to position the patient safely. -The other factors are not the prioriy

During your morning assessment of a patient with cirrhosis, you note the patient is disoriented to person and place. In addition while assessing the upper extremities, the patient's hands demonstrate a flapping motion. What lab result would explain these abnormal assessment findings? A. Decreased magnesium level B. Increased calcium level C. Increased ammonia level D. Increased creatinine level

C. Increased ammonia level Based on the assessment findings and the fact the patient has cirrhosis, the patient is experiencing hepatic encephalopathy. This is due to the buildup of toxins in the blood, specifically ammonia. The flapping motion of the hands is called "asterixis". Therefore, an increased ammonia level would confirm these abnormal assessment findings.

A 55 year old male patient is diagnosed with chronic kidney disease. The patient's recent GFR was 25 mL/min. What stage of chronic kidney disease is this known as? A. Stage 1 B. Stage 3 C. Stage 4 D. Stage 5

C. Stage 4 This is known as Stage 4 of CKD because the GFR (glomerular filtration rate) for this stage is 15-29 mL/min (patient's GFR is 25 mL/min). The other stage's criteria are as follows: Stage 1: Kidney damage with normal renal function GFR >90 ml/min but with proteinuria (3 months or more); Stage 2: Kidney damage with mild loss of renal function GFR 60-89 ml/min with proteinuria (3 months or more); Stage 3: Mild-to-severe loss of renal function GFR 30-59 mL/min; Stage 4: Severe loss renal function GFR 15-29 mL/min; Stage 5: End stage renal disease GRF less 15 mL/min

A patient diagnosed with hepatitis develops splenomegaly. When reviewing the laboratory report, which of the following results will the healthcare provider anticipate? A. Leukocytosis B. Neutrophilia C. Thrombocytopenia D. Polycythemia

C. Thrombocytopenia

When providing dietary counseling for the parents of a child diagnosed with celiac disease, the healthcare provider should include which of the following information in the teaching plan? A. "Instead of breads you may serve pasta products." B. "Avoid starchy vegetables like peas and potatoes." C. "Wheat, corn, and rice should be avoided." D. "Be sure to read all food labels carefully."

D. "Be sure to read all food labels carefully."

During a home health visit, you are assessing how a patient takes the prescribed pancreatic enzyme. The patient is unable to swallow the capsule whole, so they open the capsule and mix the beads inside the capsule with food/drink. Which food or drink is safe for the patient to mix the beads with? A. Pudding B. Ice cream C. Milk D. Applesauce

D. Applesauce

A 52-year-old man was referred to the clinic due to increased abdominal girth. He is diagnosed with ascites by the presence of a fluid thrill and shifting dullness on percussion. After administering diuretic therapy, which nursing action would be most effective in ensuring safe care? A. Measuring serum potassium for hyperkalemia B. Assessing the client for hypervolemia C. Measuring the client's weight weekly D. Documenting precise intake and output

D. Documenting precise intake and output For the client with ascites receiving diuretic therapy, careful intake and output measurement is essential for safe diuretic therapy. Diuretics lead to fluid losses, which if not monitored closely and documented, could place the client at risk for serious fluid and electrolyte imbalances. Hypokalemia, not hyperkalemia, commonly occurs with diuretic therapy. Because urine output increases, a client should be assessed for hypovolemia, not hypervolemia. Weights are also an accurate indicator of fluid balance. However, for this client, weights should be obtained daily, not weekly

A nurse is preparing to hang the initial bag of the parenteral nutrition (PN) solution via the central line of a malnourished client. The nurse ensure the availability of which medical equipment before hanging the solution? A. Glucometer. B. Dressing tray. C. Nebulizer. D. Infusion pump.

D. Infusion pump. The nurse should prepare an infusion pump prior hanging a parenteral solution. The use of an infusion pump is important to make sure that the solution does not infuse too quickly or delayed since the parenteral nutrition has a high glucose content. Option A: A glucometer is also needed since the client's glucose level is monitored every 4 to 6 hours, but it is not an essential item needed. Options B and C are not used before hanging a PN solution.

A 17-year-old patient with a leg fracture is scheduled for surgery. She reports that she is living with a friend and is an emancipated minor. She has a statement from the court for verification. Which intervention is most appropriate? a. Witness the permit after consent is obtained by the surgeon. b. Call a parent or legal guardian to sign the permit, since the patient is under 18. c. Obtain verbal consent, since written consent is not necessary for emancipated minors. d. Investigate your state's nurse practice act related to emancipated minors and informed consent.

a. Witness the permit after consent is obtained by the surgeon. An emancipated minor may sign his or her own permit. The nurse should be available to witness the signature, but no further action is required.

The nurse asks a preoperative patient to sign a surgical consent form as specified by the surgeon and then signs the form after the patient does so. By this action, what is the nurse doing? a. Witnessing the patient's signature b. Obtaining informed consent from the patient for the surgery c. Verifying that the consent for surgery is truly voluntary and informed d. Ensuring that the patient is mentally competent to sign the consent form

a. Witnessing the patient's signature The health care provider is ultimately responsible for obtaining informed consent. However, the nurse may be responsible for obtaining and witnessing the patient's signature on the consent form. The nurse may be a patient advocate during the signing of the consent form, verifying that consent is voluntary and that the patient understands the implications of consent, but the primary legal action by the nurse is witnessing the patient's signature.

What is the rationale for using preoperative checklists on the day of surgery? a. The patient is correctly identified. b. All preoperative orders and procedures have been carried out and records are complete. c. Patients' families have been informed as to where they can accompany and wait for patients. d. Preoperative medications are the last procedure before the patient is transported to the operating room.

b. All preoperative orders and procedures have been carried out and records are complete Preoperative checklists are a tool used to ensure that the many preparations and precautions performed before surgery have been completed and documented. Patient identification, instructions to the family, and administration of preoperative medications are often documented on the checklist, which ensures that no details are omitted.

When the nurse asks a preoperative patient about allergies, the patient reports a history of seasonal environmental allergies and allergies to a variety of fruits. What should the nurse do next? a. Note this information in the patient's record as hay fever and food allergies. b. Place an allergy alert wristband that identifies the specific allergies on the patient. c. Ask the patient to describe the nature and severity of any allergic responses experienced from these agents. d. Notify the anesthesia care provider (ACP) because the patient may have an increased risk for allergies to anesthetics.

c. Ask the patient to describe the nature and severity of any allergic responses experienced from these agents. Risk factors for latex allergies include a history of hay fever and allergies to foods such as avocados, kiwi, bananas, potatoes, peaches, and apricots. When a patient identifies such allergies, the patient should be further questioned about exposure to latex and specific reactions to allergens. A history of any allergic responsiveness increases the risk for hypersensitivity reactions to drugs used during anesthesia but the hay fever and fruit allergies are specifically related to latex allergy. After identifying the allergic reaction, the anesthesia care provider (ACP) should be notified, the allergy alert wristband should be applied, and the note in the record will include the allergies and reactions as well as the nursing actions related to the allergies.

A patient is scheduled for surgery requiring general anesthesia at an ambulatory surgical center. The nurse asks him when he ate last. He replies that he had a light breakfast a couple of hours before coming to the surgery center. What should the nurse do first? a.Tell the patient to come back tomorrow, since he ate a meal. b. Proceed with the preoperative checklist, including site identification. c. Notify the anesthesia care provider of when and what the patient last ate. d. Have the patient void before administering any preoperative medications

c. Notify the anesthesia care provider of when and what the patient last ate. The nothing by mouth (NPO) protocol of each surgical facility should be followed. -Restriction of fluids and food is designed to minimize the potential risk of pulmonary aspiration and to decrease the risk of postoperative nausea and vomiting. -If a patient has not followed the NPO instructions, surgery may be delayed or canceled. -The nurse should notify the anesthesia care provider immediately.

As the nurse prepares a patient the morning of surgery, the patient refuses to remove a wedding ring, saying, "I have never taken it off since the day I was married." Which response by the nurse is best? a. Have the patient sign a release and leave the ring on. b. Tape the wedding ring securely to the patient's finger. c. Tell the patient that the hospital is not liable for loss of the ring. d. Suggest that the patient give the ring to a family member to keep.

d. Suggest that the patient give the ring to a family member to keep.

The nurse interviews a patient scheduled to undergo general anesthesia for a hernia repair. Which information is most important to communicate to the surgeon and anesthesiologist before surgery? a. The patient drinks 3 or 4 cups of coffee every morning before going to work. b. The patient takes a baby aspirin daily but stopped taking aspirin 10 days ago. c. The patient drank 4 ounces of apple juice 3 hours before coming to the hospital. d. The patient's father died after receiving general anesthesia for abdominal surgery.

d. The patient's father died after receiving general anesthesia for abdominal surgery.

A client receiving hemodialysis treatment arrives at the hospital with a blood pressure of 200/100, a heart rate of 110, and a respiratory rate of 36. Oxygen saturation on room air is 89%. He complains of shortness of breath, and +2 pedal edema is noted. His last hemodialysis treatment was yesterday. Which of the following interventions should be done first? 1. Administer oxygen 2. Elevate the foot of the bed 3. Restrict the client's fluids 4. Prepare the client for hemodialysis.

1. Administer oxygen Airway and oxygenation are always the first priority. Because the client is complaining of shortness of breath and his oxygen saturation is only 89%, the nurse needs to try to increase his levels by administering oxygen. Option B: The foot of the bed may be elevated to reduce edema, but this isn't the priority. Options C and D: The client is in pulmonary edema from fluid overload and will need to be dialyzed and have his fluids restricted, but the first interventions should be aimed at the immediate treatment of hypoxia.

You are developing a care plan on Sally, a 67 y.o. patient with hepatic encephalopathy. Which of the following do you include? 1. Administering a lactulose enema as ordered. 2. Encouraging a protein-rich diet. 3. Administering sedatives, as necessary. 4. Encouraging ambulation at least four times a day.

1. Administering a lactulose enema as ordered. You may administer the laxative lactulose to reduce ammonia levels in the colon.

Hepatic encephalopathy develops when the blood level of which substance increases? 1. Ammonia 2. Amylase 3. Calcium 4. Potassium

1. Ammonia Ammonia levels increase d/t improper shunting of blood, causing ammonia to enter systemic circulation, which carries it to the brain.

A patient with ESRD has an arteriovenous fistula in the left arm for hemodialysis. Which intervention do you include in his plan of care? 1. Apply pressure to the needle site upon discontinuing hemodialysis 2. Keep the head of the bed elevated 45 degrees 3. Place the left arm on an arm board for at least 30 minutes 4. Keep the left arm dry

1. Apply pressure to the needle site upon discontinuing hemodialysis Apply pressure when discontinuing hemodialysis and after removing the venipuncture needle until all the bleeding has stopped. Bleeding may continue for 10 minutes in some patients.

You're caring for Betty with liver cirrhosis. Which of the following assessment findings leads you to suspect hepatic encephalopathy in her? 1. Asterixis 2. Chvostek's sign 3. Trousseau's sign 4. Hepatojugular reflex

1. Asterixis Asterixis is an early neurologic sign of hepatic encephalopathy elicited by asking the patient to hold her arms stretched out. Asterixis is present if the hands rapidly extend and flex.

The dialysis solution is warmed before use in peritoneal dialysis primarily to: 1. Encourage the removal of serum urea. 2. Force potassium back into the cells. 3. Add extra warmth into the body. 4. Promote abdominal muscle relaxation.

1. Encourage the removal of serum urea. The main reason for warming the peritoneal dialysis solution is that the warm solution helps dilate peritoneal vessels, which increases urea clearance. Options B and D: The warmed solution does not force potassium into the cells or promote abdominal muscle relaxation. Option C: Warmed dialyzing solution also contributes to client comfort by preventing chilly sensations, but this is a secondary reason for warming the solution.

Which of the following is the most significant sign of peritoneal infection? 1. Cloudy dialysate fluid 2. Swelling in the legs 3. Poor drainage of the dialysate fluid 4. Redness at the catheter insertion site

1. Cloudy dialysate fluid Cloudy drainage indicates bacterial activity in the peritoneum. Other signs and symptoms of infection are fever, hyperactive bowel sounds, and abdominal pain. Option B: Swollen legs may be indicative of congestive heart failure. Option C: Poor drainage of dialysate fluid is probably the result of a kinked catheter. Option D: Redness at the insertion site indicates local infection, not peritonitis. However, a local infection that is left untreated can progress to the peritoneum.

After the first hemodialysis treatment, your patient develops a headache, hypertension, restlessness, mental confusion, nausea, and vomiting. Which condition is indicated? 1. Disequilibrium syndrome 2. Respiratory distress 3. Hypervolemia 4. Peritonitis

1. Disequilibrium syndrome Disequilibrium occurs when excess solutes are cleared from the blood more rapidly than they can diffuse from the body's cells into the vascular system.

You're caring for Jane, a 57 y.o. patient with liver cirrhosis who developed ascites and requires paracentesis. Before her paracentesis, you instruct her to: 1. Empty her bladder. 2. Lie supine in bed. 3. Remain NPO for 4 hours. 4. Clean her bowels with an enema.

1. Empty her bladder. A full bladder can interfere with paracentesis and be punctured inadvertently.

he nurse assesses the client who has chronic renal failure and notes the following: crackles in the lung bases, elevated blood pressure, and weight gain of 2 pounds in one day. Based on these data, which of the following nursing diagnoses is appropriate? 1. Excess fluid volume related to the kidney's inability to maintain fluid balance. 2. Increased cardiac output related to fluid overload. 3. Ineffective tissue perfusion related to interrupted arterial blood flow. 4. Ineffective Therapeutic Regimen Management related to lack of knowledge about therapy.

1. Excess fluid volume related to the kidney's inability to maintain fluid balance. Crackles in the lungs, weight gain, and elevated blood pressure are indicators of excess fluid volume, a common complication in chronic renal failure. The client's fluid status should be monitored carefully for imbalances on an ongoing basis.

The client being seen in a physician's office has just been scheduled for a barium swallow the next day. The nurse writes down which of the following instructions for the client to follow before the test? 1. Fast for 8 hours before the test 2. Eat a regular supper and breakfast 3. Continue to take all oral medications as scheduled. 4. Monitor own bowel movement pattern for constipation

1. Fast for 8 hours before the test A barium swallow is an x-ray study that uses a substance called barium for contrast to highlight abnormalities in the GI tract. The client should fast for 8 to 12 hours before the test, depending on the physician instructions. Most oral medications also are withheld before the test. After the procedure the nurse must monitor for constipation, which can occur as a result of the presence of barium in the GI tract.

A patient with chronic alcohol abuse is admitted with liver failure. You closely monitor the patient's blood pressure because of which change that is associated with the liver failure? 1. Hypoalbuminemia 2. Increased capillary permeability 3. Abnormal peripheral vasodilation 4. Excess rennin release from the kidneys

1. Hypoalbuminemia Blood pressure decreases as the body is unable to maintain normal oncotic pressure with liver failure, so patients with liver failure require close blood pressure monitoring. Increased capillary permeability, abnormal peripheral vasodilation, and excess rennin released from the kidney's aren't direct ramifications of liver failure.

The client with chronic renal failure tells the nurse he takes magnesium hydroxide (milk of magnesia) at home for constipation. The nurse suggests that the client switch to psyllium hydrophilic mucilloid (Metamucil) because: 1. MOM can cause magnesium toxicity 2. MOM is too harsh on the bowel 3. Metamucil is more palatable 4. MOM is high in sodium

1. MOM can cause magnesium toxicity Magnesium is normally excreted by the kidneys. When the kidneys fail, magnesium can accumulate and cause severe neurologic problems. Option B: MOM is harsher than Metamucil, but magnesium toxicity is a more serious problem. Option C: A client may find both MOM and Metamucil unpalatable. Option D: MOM is not high in sodium.

The student nurse is teaching the family of a patient with liver failure. You instruct them to limit which foods in the patient's diet? 1. Meats and beans. 2. Butter and gravies. 3. Potatoes and pastas. 4. Cakes and pastries.

1. Meats and beans. Meats and beans are high-protein foods. In liver failure, the liver is unable to metabolize protein adequately, causing protein by-products to build up in the body rather than be excreted.

The nurse is performing an assessment on a client who has returned from the dialysis unit following hemodialysis. The client is complaining of a headache and nausea and is extremely restless. Which of the following is the most appropriate nursing action? 1. Notify the physician 2. Monitor the client 3. Elevate the head of the bed 4. Medicate the client for nausea

1. Notify the physician Disequilibrium syndrome may be due to the rapid decrease in BUN levels during dialysis. These changes can cause cerebral edema that leads to increased intracranial pressure. The client is exhibiting early signs of disequilibrium syndrome and appropriate treatments with anticonvulsant medications and barbiturates may be necessary to prevent a life-threatening situation. The physician must be notified

You're developing the plan of care for a patient experiencing dumping syndrome after a Billroth II procedure. Which dietary instructions do you include? 1. Omit fluids with meals. 2. Increase carbohydrate intake. 3. Decrease protein intake. 4. Decrease fat intake.

1. Omit fluids with meals. Gastric emptying time can be delayed by omitting fluids from your patient's meal. A diet low in carbs and high in fat & protein is recommended to treat dumping syndrome.

Dialysis allows for the exchange of particles across a semipermeable membrane by which of the following actions? 1. Osmosis and diffusion 2. Passage of fluid toward a solution with a lower solute concentration 3. Allowing the passage of blood cells and protein molecules through it. 4. Passage of solute particles toward a solution with a higher concentration.

1. Osmosis and diffusion Osmosis allows for the removal of fluid from the blood by allowing it to pass through the semipermeable membrane to an area of high concentrate (dialysate), and diffusion allows for passage of particles (electrolytes, urea, and creatinine) from an area of higher concentration to an area of lower concentration. Option B: Fluid passes to an area with a higher solute concentration. Option C: The pores of a semipermeable membrane are small, thus preventing the flow of blood cells and protein molecules through it.

Sharon has cirrhosis of the liver and develops ascites. What intervention is necessary to decrease the excessive accumulation of serous fluid in her peritoneal cavity? 1. Restrict fluids 2. Encourage ambulation 3. Increase sodium in the diet 4. Give antacids as prescribed

1. Restrict fluids Restricting fluids decrease the amount of body fluid and the accumulation of fluid in the peritoneal space.

A client has a percutaneous endoscopic gastrostomy tube inserted for tube feedings. Before starting a continuous feeding, the nurse should place the client in which position? 1. Semi-Fowlers 2. Supine 3. Reverse Trendelenburg 4. High Fowler's

1. Semi-Fowlers To prevent aspiration of stomach contents, the nurse should place the client in semi-Fowler's position. High Fowler's position isn't necessary and may not be tolerated as well as semi-Fowler's.

A client with chronic renal failure has asked to be evaluated for a home continuous ambulatory peritoneal dialysis (CAPD) program. The nurse should explain that the major advantage of this approach is that it: 1. Is relatively low in cost 2. Allows the client to be more independent 3. Is faster and more efficient than standard peritoneal dialysis 4. Has fewer potential complications than standard peritoneal dialysis

2. Allows the client to be more independent The major benefit of CAPD is that it frees the client from daily dependence on dialysis centers, home health care personnel, and machines for life-sustaining treatment. The independence is a valuable outcome for some people. Option A: CAPD is costly and must be done daily. Option D: Side effects and complications are similar to those of standard peritoneal dialysis.

A 30 y.o. female patient is undergoing hemodialysis with an internal arteriovenous fistula in place. What do you do to prevent complications associated with this device? 1. Insert I.V. lines above the fistula. 2. Avoid taking blood pressures in the arm with the fistula. 3. Palpate pulses above the fistula. 4. Report a bruit or thrill over the fistula to the doctor.

2. Avoid taking blood pressures in the arm with the fistula. Don't take blood pressure readings in the arm with the fistula because the compression could damage the fistula. Option A: IV lines shouldn't be inserted in the arm used for hemodialysis. Option C: Palpate pulses below the fistula. Option D: Lack of bruit or thrill should be reported to the doctor.

Stephanie, a 28 y.o. accident victim, requires TPN. The rationale for TPN is to provide: 1. Necessary fluids and electrolytes to the body. 2. Complete nutrition by the I.V. route. 3. Tube feedings for nutritional supplementation. 4. Dietary supplementation with liquid protein given between meals.

2. Complete nutrition by the I.V. route. TPN is given I.V. to provide all the nutrients your patient needs. TPN isn't a tube feeding nor is it a liquid dietary supplement.

The client being hemodialyzed suddenly becomes short of breath and complains of chest pain. The client is tachycardic, pale, and anxious. The nurse suspects air embolism. The nurse should: 1. Continue the dialysis at a slower rate after checking the lines for air 2. Discontinue dialysis and notify the physician 3. Monitor vital signs every 15 minutes for the next hour 4. Bolus the client with 500 ml of normal saline to break up the air embolism.

2. Discontinue dialysis and notify the physician If the client experiences air embolus during hemodialysis, the nurse should terminate dialysis immediately, notify the physician, and administer oxygen as needed.

Your patient becomes restless and tells you she has a headache and feels nauseous during hemodialysis. Which complication do you suspect? 1. Infection 2. Disequilibrium syndrome 3. Air embolus 4. Acute hemolysis

2. Disequilibrium syndrome Disequilibrium syndrome is caused by a rapid reduction in urea, sodium, and other solutes from the blood. This can lead to cerebral edema and increased intracranial pressure (ICP). Signs and symptoms include headache, nausea, restlessness, vomiting, confusion, twitching, and seizures.

You're caring for Lewis, a 67 y.o. patient with liver cirrhosis who developed ascites and requires paracentesis. Relief of which symptom indicated that the paracentesis was effective? 1. Pruritus 2. Dyspnea 3. Jaundice 4. Peripheral Neuropathy

2. Dyspnea Ascites puts pressure on the diaphragm. Paracentesis is done to remove fluid and reducing pressure on the diaphragm. The goal is to improve the patient's breathing. The others are signs of cirrhosis that aren't relieved by paracentesis.

What is the priority nursing diagnosis with your patient diagnosed with end-stage renal disease? 1. Activity intolerance 2. Fluid volume excess 3. Knowledge deficit 4. Pain

2. Fluid volume excess Fluid volume excess because the kidneys aren't removing fluid and wastes. The other diagnoses may apply, but they don't take priority.

A client with irritable bowel syndrome is being prepared for discharge. Which of the following meal plans should the nurse give the client? 1. Low fiber, low-fat 2. High fiber, low-fat 3. Low fiber, high-fat 4. High-fiber, high-fat

2. High-fiber, low-fat The client with irritable bowel syndrome needs to be on a diet that contains at least 25 grams of fiber per day. Fatty foods are to be avoided because they may precipitate symptoms.

The nurse is instructing a client with diabetes mellitus about peritoneal dialysis. The nurse tells the client that it is important to maintain the dwell time for the dialysis at the prescribed time because of the risk of: 1. Infection 2. Hyperglycemia 3. Fluid overload 4. Disequilibrium syndrome

2. Hyperglycemia An extended dwell time increases the risk of hyperglycemia in the client with diabetes mellitus as a result of absorption of glucose from the dialysate and electrolyte changes. Diabetic clients may require extra insulin when receiving peritoneal dialysis

Your patient is complaining of muscle cramps while undergoing hemodialysis. Which intervention is effective in relieving muscle cramps? 1. Increase the rate of dialysis 2. Infuse normal saline solution 3. Administer a 5% dextrose solution 4. Encourage active ROM exercises

2. Infuse normal saline solution Treatment includes administering normal saline or hypertonic normal saline solution because muscle cramps can occur when the sodium and water are removed to quickly during dialysis. Reducing the rate of dialysis, not increasing it, may alleviate muscle cramps.

The nurse has completed client teaching with the hemodialysis client about self-monitoring between hemodialysis treatments. The nurse determines that the client best understands the information given if the client states to record the daily: 1. Pulse and respiratory rate 2. Intake, output, and weight 3. BUN and creatinine levels 4. Activity log

2. Intake, output, and weight The client on hemodialysis should monitor fluid status between hemodialysis treatments by recording intake and output and measuring weight daily. Ideally, the hemodialysis client should not gain more than 0.5 kg of weight per day.

An intubated patient is receiving continuous enteral feedings through a Salem sump tube at a rate of 60ml/hr. Gastric residuals have been 30-40ml when monitored Q4H. You check the gastric residual and aspirate 220ml. What is your first response to this finding? 1. Notify the doctor immediately. 2. Stop the feeding, and clamp the NG tube. 3. Discard the 220ml, and clamp the NG tube. 4. Give a prescribed GI stimulant such as metoclopramide (Reglan).

2. Stop the feeding, and clamp the NG tube. A gastric residual greater than 2 hours worth of feeding or 100-150ml is considered too high. The feeding should be stopped; NG tube clamped, and then allow time for the stomach to empty before additional feeding is added.

A client is diagnosed with chronic renal failure and told she must start hemodialysis. Client teaching would include which of the following instructions? 1. Follow a high potassium diet 2. Strictly follow the hemodialysis schedule 3. There will be a few changes in your lifestyle. 4. Use alcohol on the skin and clean it due to integumentary changes.

2. Strictly follow the hemodialysis schedule To prevent life-threatening complications, the client must follow the dialysis schedule. Option A: The client should follow a low-potassium diet because potassium levels increase in chronic renal failure. Option C: The client should know hemodialysis is time-consuming and will definitely cause a change in current lifestyle. Option D: Alcohol would further dry the client's skin more than it already is.

In planning teaching strategies for the client with chronic renal failure, the nurse must keep in mind the neurologic impact of uremia. Which teaching strategy would be most appropriate? 1. Providing all needed teaching in one extended session. 2. Validating frequently the client's understanding of the material. 3. Conducting a one-on-one session with the client. 4. Using videotapes to reinforce the material as needed.

2. Validating frequently the client's understanding of the material. Uremia can cause decreased alertness, so the nurse needs to validate the client's comprehension frequently. Option A: Because the client's ability to concentrate is limited, short lesions are most effective. Option C: If family members are present at the sessions, they can reinforce the material. Option D: Written materials that the client can review are superior to videotapes, because the clients may not be able to maintain alertness during the viewing of the videotape.

A client is undergoing peritoneal dialysis. The dialysate dwell time is completed, and the dwell clamp is opened to allow the dialysate to drain. The nurse notes that the drainage has stopped and only 500 ml has drained; the amount the dialysate instilled was 1,500 ml. Which of the following interventions would be done first? 1. Change the client's position. 2. Call the physician. 3. Check the catheter for kinks or obstruction. 4. Clamp the catheter and instill more dialysate at the next exchange time

3. Check the catheter for kinks or obstruction. The first intervention should be to check for kinks and obstructions because that could be preventing drainage. After checking for kinks, have the client change position to promote drainage. Don't give the next scheduled exchange until the dialysate is drained because abdominal distention will occur, unless the output is within parameters set by the physician. If unable to get more output despite checking for kinks and changing the client's position, the nurse should then call the physician to determine the proper intervention.

The nurse would question an order for which type of antacid in patients with chronic renal failure? 1. Aluminum-containing antacids 2. Calcium-containing antacids 3. Magnesium-containing antacids 4. All of the above.

3. Magnesium-containing antacids Magnesium-containing antacids can cause hypermagnesemia in patients with chronic renal failure. Aluminum-containing antacids may be used as a phosphate binder in patients with chronic renal failure. Calcium-containing antacids are also appropriate because these patients may be hypocalcemic.

Your patient has a GI tract that is functioning, but has the inability to swallow foods. Which is the preferred method of feeding for your patient? 1. TPN 2. PPN 3. NG feeding 4. Oral liquid supplements

3. NG feeding Because the GI tract is functioning, feeding methods involve the enteral route which bypasses the mouth but allows for a major portion of the GI tract to be used.

Britney, a 20 y.o. student is admitted with acute pancreatitis. Which laboratory findings do you expect to be abnormal for this patient? 1. Serum creatinine and BUN 2. Alanine aminotransferase (ALT) and aspartate aminotransferase (AST) 3. Serum amylase and lipase 4. Cardiac enzymes

3. Serum amylase and lipase Pancreatitis involves activation of pancreatic enzymes, such as amylase and lipase. These levels are elevated in a patient with acute pancreatitis.

The client with chronic renal failure who is scheduled for hemodialysis this morning is due to receive a daily dose of an antihypertensive. The nurse should plan to administer this medication: 1. Just before dialysis 2. During dialysis 3. On return from dialysis 4. The day after dialysis

3. On return from dialysis Antihypertensive medications are given to the client following hemodialysis. This prevents the client from becoming hypotensive during dialysis and also from having the medication removed from the bloodstream by dialysis. No rationale exists for waiting a full day to resume the medication. This would lead to ineffective control of the blood pressure.

The client with acute renal failure has a serum potassium level of 5.8 mEq/L. The nurse would plan which of the following as a priority action? 1. Allow an extra 500 ml of fluid intake to dilute the electrolyte concentration. 2. Encourage increased vegetables in the diet 3. Place the client on a cardiac monitor 4. Check the sodium level

3. Place the client on a cardiac monitor The client with hyperkalemia is at risk for developing cardiac dysrhythmias and cardiac arrest. Because of this, the client should be placed on a cardiac monitor. Option A: Fluid intake is not increased because it contributes to fluid overload and would not affect the serum potassium level significantly. Option B: Vegetables are a natural source of potassium in the diet, and their use would not be increased. Option D: The nurse may also assess the sodium level because sodium is another electrolyte commonly measured with the potassium level. However, this is not a priority action at this time.

A patient is experiencing which type of incontinence if she experiences leaking urine when she coughs, sneezes, or lifts heavy objects? 1. Overflow 2. Reflex 3. Stress 4. Urge

3. Stress Stress incontinence is an involuntary loss of a small amount of urine due to sudden increased intra-abdominal pressure, such as with coughing or sneezing.

The nurse is monitoring a client for the early signs of dumping syndrome. Which symptom indicates this occurrence? 1. Abdominal cramping and pain 2. Bradycardia and indigestion 3. Sweating and pallor 4. Double vision and chest pain

3. Sweating and pallor Early manifestations of dumping syndrome occur 5 to 30 minutes after eating. Symptoms include vertigo, tachycardia, syncope, sweating, pallor, palpitations, and the desire to lie down.

When caring for Mr. Roberto's AV shunt on his right arm, you should: 1. Cover the entire cannula with an elastic bandage 2. Notify the physician if a bruit and thrill are present 3. User surgical aseptic technique when giving shunt care 4. Take the blood pressure on the right arm instead

3. User surgical aseptic technique when giving shunt care

When teaching an elderly client how to prevent constipation, which of the following instructions should the nurse include? 1. "Drink 6 glasses of fluid each day." 2. "Avoid grain products and nuts." 3. "Add at least 4 grams of bran to your cereal each morning." 4. "Be sure to get regular exercise.

4. "Be sure to get regular exercise." Exercise helps prevent constipation. Fluids and dietary fiber promote normal bowel function. The client should drink eight to ten glasses of fluid each day. Although adding bran to cereal helps prevent constipation by increasing dietary fiber, the client should start with a small amount and gradually increase the amount as tolerated to a maximum of 2 grams a day.

You're discharging Nathaniel with hepatitis B. Which statement suggests understanding by the patient? 1. "Now I can never get hepatitis again." 2. "I can safely give blood after 3 months." 3. "I'll never have a problem with my liver again, even if I drink alcohol." 4. "My family knows that if I get tired and start vomiting, I may be getting sick again."

4. "My family knows that if I get tired and start vomiting, I may be getting sick again." Hepatitis B can recur. Patients who have had hepatitis are permanently barred from donating blood. Alcohol is metabolized by the liver and should be avoided by those who have or had hepatitis B.

The client with chronic renal failure returns to the nursing unit following a hemodialysis treatment. On assessment the nurse notes that the client's temperature is 100.2. Which of the following is the most appropriate nursing action? 1. Encourage fluids 2. Notify the physician 3. Monitor the site of the shunt for infection 4. Continue to monitor vital signs

4. Continue to monitor vital signs The client may have an elevated temperature following dialysis because the dialysis machine warms the blood slightly. If the temperature is elevated excessively and remains elevated, sepsis would be suspected, and a blood sample would be obtained as prescribed for culture and sensitivity purposes.

Katrina is diagnosed with lactose intolerance. To avoid complications with lack of calcium in the diet, which food should be included in the diet? 1. Fruit 2. Whole grains 3. Milk and cheese products 4. Dark green, leafy vegetables

4. Dark green, leafy vegetables Dark green, leafy vegetables are rich in calcium.

The client newly diagnosed with chronic renal failure recently has begun hemodialysis. Knowing that the client is at risk for disequilibrium syndrome, the nurse assesses the client during dialysis for: 1. Hypertension, tachycardia, and fever 2. Hypotension, bradycardia, and hypothermia 3. restlessness, irritability, and generalized weakness 4. Headache, deteriorating level of consciousness, and twitching

4. Headache, deteriorating level of consciousness and twitching. Disequilibrium syndrome is characterized by headache, mental confusion, decreasing level of consciousness, nausea, and vomiting, twitching, and possible seizure activity. Disequilibrium syndrome is caused by rapid removal of solutes from the body during hemodialysis. At the same time, the blood-brain barrier interferes with the efficient removal of wastes from brain tissue. As a result, water goes into cerebral cells because of the osmotic gradient, causing brain swelling and onset of symptoms. The syndrome most often occurs in clients who are new to dialysis and is prevented by dialyzing for shorter times or at reduced blood flow rates.

The main indicator of the need for hemodialysis is: 1. Ascites 2. Acidosis 3. Hypertension 4. Hyperkalemia

4. Hyperkalemia

The nurse is reviewing a list of components contained in the peritoneal dialysis solution with the client. The client asks the nurse about the purpose of the glucose contained in the solution. The nurse bases the response knowing that the glucose: 1. Prevents excess glucose from being removed from the client. 2. Decreases risk of peritonitis. 3. Prevents disequilibrium syndrome 4. Increased osmotic pressure to produce ultrafiltration.

4. Increases osmotic pressure to produce ultrafiltration. Increasing the glucose concentration makes the solution increasingly more hypertonic. The more hypertonic the solution, the greater the osmotic pressure for ultrafiltration and thus the greater amount of fluid removed from the client during an exchange.

Your patient with chronic renal failure reports pruritus. Which instruction should you include in this patient's teaching plan? 1. Rub the skin vigorously with a towel 2. Take frequent baths 3. Apply alcohol-based emollients to the skin 4. Keep fingernails short and clean

4. Keep fingernails short and clean Calcium-phosphate deposits in the skin may cause pruritus. Scratching leads to excoriation and breaks in the skin that increase the patient's risk of infection. Keeping fingernails short and clean helps reduce the risk of infection.

Which of the following substances is most likely to cause gastritis? 1. Milk 2. Bicarbonate of soda, or baking soda 3. Enteric coated aspirin 4. Nonsteriodal anti-inflammatory drugs

4. Nonsteroidal anti-inflammatory drugs NSAIDS are a common cause of gastritis because they inhibit prostaglandin synthesis. Milk, once thought to help gastritis, has little effect on the stomach mucosa. Bicarbonate of soda, or baking soda, may be used to neutralize stomach acid, but it should be used cautiously because it may lead to metabolic acidosis. ASA with enteric coating shouldn't contribute significantly to gastritis because the coating limits the aspirin's effect on the gastric mucosa.

What is the best way to check for patency of the arteriovenous fistula for hemodialysis? 1. Pinch the fistula and note the speed of filling on release 2. Use a needle and syringe to aspirate blood from the fistula 3. Check for capillary refill of the nail beds on that extremity 4. Palpate the fistula throughout its length to assess for a thrill

4. Palpate the fistula throughout its length to assess for a thrill The vibration or thrill felt during palpation ensures that the fistula has the desired turbulent blood flow. Pinching the fistula could cause damage. Aspirating blood is a needless invasive procedure.

Leigh Ann is receiving pancrelipase (Viokase) for chronic pancreatitis. Which observation best indicates the treatment is effective? 1. There is no skin breakdown. 2. Her appetite improves. 3. She loses more than 10 lbs. 4. Stools are less fatty and decreased in frequency.

4. Stools are less fatty and decreased in frequency. Pancrelipase provides the exocrine pancreatic enzyme necessary for proper protein, fat, and carb digestion. With increased fat digestion and absorption, stools become less frequent and normal in appearance.

Your patient had surgery to form an arteriovenous fistula for hemodialysis. Which information is important for providing care for the patient? 1. The patient shouldn't feel pain during initiation of dialysis 2. The patient feels best immediately after the dialysis treatment 3. Using a stethoscope for auscultating the fistula is contraindicated 4. Taking a blood pressure reading on the affected arm can cause clotting of the fistula

4. Taking a blood pressure reading on the affected arm can cause clotting of the fistula Pressure on the fistula or the extremity can decrease blood flow and precipitate clotting, so avoid taking blood pressure on the affected arm.

A client with chronic renal failure has completed a hemodialysis treatment. The nurse would use which of the following standard indicators to evaluate the client's status after dialysis? 1. Potassium level and weight 2. BUN and creatinine levels 3. VS and BUN 4. VS and weight.

4. VS and weight. Following dialysis, the client's vital signs are monitored to determine whether the client is remaining hemodynamically stable. Weight is measured and compared with the client's predialysis weight to determine the effectiveness of fluid extraction. Options A, B, and C: Laboratory studies are done as per protocol but are not necessarily done after the hemodialysis treatment has ended.

Dr. Smith has determined that the client with hepatitis has contracted the infection from contaminated food. The nurse understands that this client is most likely experiencing what type of hepatitis? A. Hepatitis A B. Hepatitis B C. Hepatitis C D. Hepatitis D

A. Hepatitis A Hepatitis A is transmitted by the fecal-oral route via contaminated food or infected food handlers. Hepatitis B, C, and D are transmitted most commonly via infected blood or body fluids.

The nurse instructs a client with renal failure who is receiving hemodialysis about dietary modifications. The nurse determines that the client understands these dietary modifications if the client selects which items from the dietary menu? A. Mushroom and blueberry. B. Beans and banana. C. Fish and tomato juice. D. Potato and spinach.

A. Mushroom and blueberry. A renal diet is one that is low in sodium, phosphorous, potassium and protein. Options B, C, and D are high in sodium, phosphorus, and potassium.

A nurse is preparing to hang a fat emulsion (lipids) and observes some visible fat globules at the top of the solution. The nurse ensure to do which of the following actions? A. Take another bottle of solution. B. Runs the bottle solution under a warm water. C. Rolls the bottle solution gently. D. Shake the bottle solution vigorously.

A. Take another bottle of solution. Fat emulsions are used as dietary supplements for patients who are unable to get enough fat in their diet, usually because of certain illnesses or recent surgery. The nurse should examine the bottle of fat emulsion for separation of emulsion into layers or fat globules or the accumulation of froth. The nurse should not hang a fat emulsion if any of these observed and should return the solution to the pharmacy.

The nurse is assisting a client on a low-potassium diet to select food items from the menu. Which of the following food items, if selected by the client, would indicate an understanding of this dietary restriction? 1. Cantaloupe 2. Spinach 3. Lima beans 4. Strawberries

Answer: 3. Lima beans Lima beans (1/3 c) averages three (3) mEq per serving. Option A: Cantaloupe (1/4 small) Option B: Spinach (1/2 cooked) Option D: Strawberries (1 ¼ cups) are high potassium foods and average 7 mEq per serving.

A client receiving parenteral nutrition (PN) complains of a headache. A nurse notes that the client has a bounding pulse, jugular distension, and weight gain greater than desired. The nurse determines that the client is experiencing which complication of PN therapy? A. Air embolism. B. Hypervolemia. C. Hyperglycemia. D. Sepsis.

B. Hypervolemia. The client's sign and symptoms are consistent with hypervolemia. This happen when the client receives excessive fluid administration or administration of fluid too rapidly.

The nurse is teaching a client who has iron deficiency anemia about foods she should include in her diet. The nurse determines that the client understands the dietary instructions if she selects which of the following from her menu? A. Nuts and fish. B. Oranges and dark green leafy vegetables. C. Butter and margarine. D. Sugar and candy.

B. Oranges and dark green leafy vegetables. Dark green leafy vegetables are rich in iron while oranges are a good source of vitamin C, which enhances iron absorption.

A stroke client who was initially on NGT feeding was able to tolerate soft diet so the physician ordered for the removal of it. The nurse would instruct the client to do which of the following before he removes the tube? A. Inhale and exhale simultaneously. B. Take a long breath and hold it. C. Do a Valsalva maneuver. D. Blow the nose.

B. Take a long breath and hold it. Holding the breath closes the glottis hence it will be easier to withdraw the tube through the esophagus into the nose. and this method will also prevent aspiration.

Nurse Berlinda is assigned to a 41-year-old client who has a diagnosis of chronic pancreatitis. The nurse reviews the laboratory result, anticipating a laboratory report that indicates a serum amylase level of: A. 45 units/L B. 100 units/L C. 300 units/L D. 500 units/L

C. 300 units/L The normal serum amylase level is 25 to 151 units/L. With chronic cases of pancreatitis, the rise in serum amylase levels usually does not exceed three times the normal value. In acute pancreatitis, the value may exceed five times the normal value. Options A and B are within normal limits. Option D is an extremely elevated level seen in acute pancreatitis.

A patient receiving parenteral nutrition is administered via the following routes except: A. Subclavian line. B. Central Venous Catheter. C. PICC (Peripherally inserted central catheter) line. D. PEG tube.

D. PEG tube. Percutaneous endoscopic gastrostomy (PEG tube) is inserted into a person's stomach through the abdominal wall that is used to provide a means of feeding when oral intake is not adequate. While Parenteral nutrition bypasses the digestive system by the administration to the bloodstream.

The nurse is preparing to care for a client receiving peritoneal dialysis. Which of the following would be included in the nursing plan of care to prevent the major complication associated with peritoneal dialysis? 1. Monitor the client's level of consciousness 2. Maintain strict aseptic technique 3. Add heparin to the dialysate solution 4. Change the catheter site dressing daily

2. Maintain strict aseptic technique The major complication of peritoneal dialysis is peritonitis. Strict aseptic technique is required in caring for the client receiving this treatment. Although option D may assist in preventing infection, this option relates to an external site.

Which of the following nursing interventions should be included in the client's care plan during dialysis therapy? 1. Limit the client's visitors 2. Monitor the client's blood pressure 3. Pad the side rails of the bed 4. Keep the client NPO.

2. Monitor the client's blood pressure Because hypotension is a complication of peritoneal dialysis, the nurse records intake, and output, monitors VS, and observes the client's behavior. Option A: The nurse also encourages visiting and other diversional activities. Options C and D: A client on PD does not need to be placed in bed with padded side rails or kept NPO.

A nurse is assessing the patency of an arteriovenous fistula in the left arm of a client who is receiving hemodialysis for the treatment of chronic renal failure. Which finding indicates that the fistula is patent? 1. Absence of bruit on auscultation of the fistula. 2. Palpation of a thrill over the fistula 3. Presence of a radial pulse in the left wrist 4. Capillary refill time less than 3 seconds in the nail beds of the fingers on the left hand.

2. Palpation of a thrill over the fistula The nurse assesses the patency of the fistula by palpating for the presence of a thrill or auscultating for a bruit. Option A: The presence of a thrill and bruit indicate patency of the fistula. Options C and D: Although the presence of a radial pulse in the left wrist and capillary refill time less than 3 seconds in the nail beds of the fingers on the left hand are normal findings; they do not assess fistula patency.

A nurse is making initial rounds at the beginning of the shift and notice that the parenteral nutrition (PN) bag of an assigned client is empty. Which of the following solutions readily available on the nursing unit should the nurse hang until another PN solution is mixed and delivered to the nursing unit? A. 10% dextrose in water. B. 5% dextrose in water. C. 5% dextrose in normal saline. D. 5% dextrose in lactated Ringer solution.

A. 10% dextrose in water. The client is at risk of hypoglycemia. Hence the nurse will hang a solution that has the highest amount of glucose until the new parenteral nutrition solution becomes readily available.

Nurse Ryan is assessing for correct placement of a nasogastric tube. The nurse aspirates the stomach contents and checks the contents for pH. The nurse verifies correct tube placement if which pH value is noted? A. 3.5 B. 7.0 C. 7.35 D. 7.5

A. 3.5 If the nasogastric tube is in the stomach, the pH of the contents will be acidic. Gastric aspirates have acidic pH values and should be 3.5 or lower. Option B indicates a slightly acidic pH. Option C indicates a neutral pH. Option D indicates an alkaline pH

While palpating a female client's right upper quadrant (RUQ), the nurse would expect to find which of the following structures? A. Sigmoid colon B. Appendix C. Spleen D. Liver

D. Liver The RUQ contains the liver, gallbladder, duodenum, head of the pancreas, hepatic flexure of the colon, portions of the ascending and transverse colon, and a portion of the right kidney. The sigmoid colon is located in the left lower quadrant; the appendix, in the right lower quadrant; and the spleen, in the left upper quadrant.

A nurse is inserting a nasogastric tube in an adult male client. During the procedure, the client begins to cough and has difficulty breathing. Which of the following is the appropriate nursing action? A. Quickly insert the tube B. Notify the physician immediately C. Remove the tube and reinsert when the respiratory distress subsides D. Pull back on the tube and wait until the respiratory distress subsides

D. Pull back on the tube and wait until the respiratory distress subsides During the insertion of a nasogastric tube, if the client experiences difficulty breathing or any respiratory distress, withdraw the tube slightly, stop the tube advancement, and wait until the distress subsides. Options B and C are unnecessary. Quickly inserting the tube is not an appropriate action because, in this situation, it may be likely that the tube has entered the bronchus.


Related study sets

NU371 Week 6 PrepU: Pancreatitis

View Set

Prepu: Chapter 64: Introduction to the Integumentary System (Donnelly-Moreno & Moseley, 2022)

View Set

Applied Leadership Principles Chapter 1, 2, 3, 4

View Set

Pharmacology ATI FINAL EXAM PART 2

View Set

Poetic Devices in "A Consumer's Report"

View Set

Chapter 11: Deliver the goods: determine the distribution strategy

View Set